LSAT and Law School Admissions Forum

Get expert LSAT preparation and law school admissions advice from PowerScore Test Preparation.

User avatar
 Dave Killoran
PowerScore Staff
  • PowerScore Staff
  • Posts: 5862
  • Joined: Mar 25, 2011
|
#92217
Complete Question Explanation
(The complete setup for this game can be found here: lsat/viewtopic.php?f=371&t=15040)

The correct answer choice is (D).

This is an easy List question—apply correct List question technique and you can solve the question quite quickly.

Answer choice (A) is incorrect because it violates the fifth rule.

Answer choice (B) is incorrect because it violates the fifth rule.

Answer choice (C) is incorrect because it violates the fourth rule.

Answer choice (D) is the correct answer.

Answer choice (E) is incorrect because it violates the first rule as there is no officer assigned to the committee.

Get the most out of your LSAT Prep Plus subscription.

Analyze and track your performance with our Testing and Analytics Package.